You are on page 1of 7

Math 426: Homework 4

Mary Radcliffe
due 2 May 2014

In Bartle:
5C. If f L(X, F, ) and g is a F-measurable real-valued function
such
R
R that
f (x) = g(x) almost everywhere, then g L(X, F, ) and f d = g d.
+
Proof. Let h(x) = |f (x) g(x)|. Then h M
R (X, F), and h = 0 almost
everywhere. Therefore, h is integrable and h d = 0. But
R this implies

(f g) d
that
f
g
is
integrable,
and
thus
g
is
integrable.
Moreover,
R
R
h d = 0, and thus (f g) d = 0. The result immediately follows by
linearity of integration.

5E. If f L and g is a bounded, measurable function, then f g L.


Proof. Let M be such that |g| M . Note that by linearity of integration,
we have that M f L, and |f g| |M f | for all x. Then by Corollary 5.4,
f g is integrable.
5I. If f is a complex-valued function on X such that ReR f and ImR f belong to
L(X,
R F, ), we say that f is integrable and define f d = Re f d +
i Im f d. Let f be a complex-valued measurable function. Show
that

R
f d
Rf is integrable if and only if |f | is integrable, in which case
|f | d.
Proof. Let us write f = f1 + if2 , so that f1 = Re f and f2 = Im f . We
have that f is measurable if and only if f1 and f2 are measurable, and
1/2
|f | = (f1 )2 + (f2 )2
. Thus, f is measurable implies that |f | is also
measurable.
p
p

Notice, if a, b R, then a2 + b2 =
(a + b)2 2ab
(a + b)2 =
|a + b| |a| + |b|. Therefore, if f is integrable, then |f | is a measurable
function such that |f | |f1 | + |f2 |, and thus by Corollary 5.4, |f | is
integrable.
Conversely, if |f | is integrable, note that |f1 | < |f | and |f2 | < |f |, so we
have f1 and f2 are real-valued integrable functions by Corollary 5.4. But
then by definition f is also integrable.
R

R
For the inequality, write f d = rei , so thatR f d R= r. Let g(x) =
ei fR(x), so that g(x) = g1 (x)R + ig2 (x), and g d = ei f (x) d =
ei f (x) d = r. But then g2 d = 0, so g2 = 0 almost everywhere.
Moreover, |f | = |ei ||f | = |g| = |g1 | almost everywhere.
Therefore, we have
Z

Z
Z
Z
Z




f d = r = g1 d = g1 d |g1 | d = |f | d,




1

since g1 is real valued and we may use Theorem 5.3.

5P. Let fn L(X, F,


R ), and suppose that R{fn } convergesR to a function f .
Show that if lim |fn f | d = 0, then |f | d = lim |fn | d.
Proof. Note that as |fn f | is integrable for n sufficiently large, we have
fn f is integrable, and thus by linearity f is integrable. Moreover, for
all n, we have |fn | |fn f | + |f |, so |fnR| |f | |fnR f |. Integrating
R and
taking limits on both sides,
we
have
lim
|f
|
d

|f
|
d

lim
|fn
n
R
R
f | d = 0, and thus lim |fn | d = |f | d
R
5Q. If t > 0, then 0 etx dx = 1t . Moreover, if t a > 0, then etx eax .
Use this and Exercise 4M to
justify differentiating under the integral sign
R
and to obtain the formula 0 xn ex dx = n!.
Proof. Now,
Z

etx dx

Z
=

lim

etx dx

b
1 tx
lim e
b
t
x=0


1 tb 1
lim e
+
b
t
t
1
,
t

=
=
=

where the integral is obtained by the fundamental theorem of calculus.


tx
, so | f
Let f (x, t) = etx for t [ 21 , 32 ]. Then f
t (x, t) = xe
t (x, t)|
x
x

xe 2 for all t. Let g(x) = xe 2 . We claim that g is integrable on [0, ).


This can be seen in several ways. Note that g is measurable. Moreover,
note that if x is sufficiently large (in fact, x > 8 ln 4 is sufficient), we
x
have that x < ex/4 . Define h(x) = g(x) if x 8 ln 4 and h(x) = e 4 if
x > 8 ln 4. Then g h, and h is clearly integrable, and thus by Corollary
5.4, g is also integrable. But then we have, by Theorem 5.9, that
 
Z

1
d 1
d
tx
2 =
=
e dx
t
dt t
dt
0
!
Z
d
tx
e d(x)
=
dt
[0,)
Z
tx 
=
e
d(x)
[0,) t
Z
=
xetx d(x)
[0,)
Z

xetx d(x).

Taking t = 1 and multiplying by 1 yields the result for n = 1.


R
Now, proceed by induction on n. Suppose it is known that 0 xn1 etx dx =
(n1)!
Let fn (x, t) = xn1 etx for t [ 21 , 32 ]. As above, we have
tn .
fn
n tx
n x
n
2.
, so | f
Again, for x sufficiently
t (x, t) = x e
t (x, t)| x e

large, we have that xn < ex/4 , so as above, we have that xn e 2 is integrable on [0, ). Then as above, we have


Z

d (n 1)!
d
n!
n1 tx
=
x
e
dx
n+1 =
t
dt
tn
dt
0
Z
n1 tx
=
(x
e )d(x)
[0,) t
Z
xn etx dx.
=
0

Taking t = 1 and multiplying by 1 yields the result for n.

5R. Suppose that f is defined on X [a, b] to R and that the function x 7


f (x, t) is F-measurable for each t [a, b]. Suppose that for some t0 , t1
[a, b], the function x 7 f (x, t0 ) is integrable on X, that f
t (x, t1 ) exists,

(x,t1 )
and that there exists an integrable function g on X such that f (x,t)f

tt1
g(x) for x X and t [a, b], t 6= t1 . Then
 Z

Z
d
f
=
f (x, t) d(x)
(x, t1 ) d(x).
dt
t
t=t1
Proof. Choose tn to be any sequence with tn t1 , tn 6= t1 (we may
(x,t1 )
start at n = 2 to avoid confusion). Put hn (x) = f (x,tntn)f
, so by
t1
f
hypothesis |hn (x)| g(x) for all x, and hn (x) tR(x, t1 ). By the
dominated convergence theorem, then, we have limn X hn (x)d(x) =
R f
(x, t1 )d(x). On the other hand,
X t
Z
Z
f (x, tn ) f (x, t1 )
lim
hn (x)d(x) = lim
d(x)
n X
n
tn t1
Z

Z
1
= lim
f (x, tn )d(x) f (x, t1 )d(x)
n tn t1
 Z

d
,
=
f (x, t)d(x)
dt
t=t1

as desired.
5T. Let f be a F-measurable function on X to R. For n Z+ , let {fn }
be theR sequence of Rtruncates of f . If f is integrable
with respect to ,
R
then f d = lim fn d. Conversely, if sup |fn | d < , then f is
integrable.
Proof. Note that for all
R n, we have
R |fn | f , and thus if f is integrable,
the DCT implies that fn d f d.
For the converse, note that |fn | is a monotonically increasing sequence
of
R nonnegative
R functions with limit |f |, and Rthus the MCT implies that
|fn | d |f | d. Thus, by hypothesis, |f | d < , and therefore
|f | and thus f are integrable.
4O. Fatous Lemma has an extension to a case where
R the fn take on negative
values. Let h M + (X, F), and supposeR that h d < . If {f
R n } is a
sequence in M (X, F) and h fn then lim inf fn d lim inf fn d.
3

Proof. Note that fn + h 0. Thus we have


Z
Z
Z
lim inf fn d + h d =
(lim inf fn + lim inf h) d
Z
=
(lim inf(fn + h)) d
Z
lim inf (fn + h) d

Z
Z
fn d + h d
= lim inf
Z
Z
= lim inf fn d + h d.
Subtracting

h d yields the result.

Also, complete the following:


1. Compute the following limits. Justify each computational step using convergence theorems and/or calculus.
R
(a) limn 0 1+nn2 x2 dx.
n
R
(b) limn 0 1 + nx
log(2 + cos( nx ))dx

Rn
(c) limn n f 1 + nx2 g(x)dx, where g : R R is (Lebesgue) integrable and f : R R is bounded, measurable, and continuous at
1.
Solution. (a) We consider this as two integrals, over [0, 1] and [1, ).
Note that on [0, 1], we have
Z 1
n
1
dx = lim [arctan(nx)]0
lim
n
n 0 1 + n2 x2

= lim arctan(n) = .
n
2
For the other part, notice that 1+nn2 x2 n2nx2 nx1 2 x12 , and
R
moreover, 1 x12 = 1 < , so x12 L([1, ), L, ). Therefore, by
the dominated convergence theorem, we have
Z
Z
n
n
lim
dx
=
lim
d(x)
2
2
2 2
n 1
n
1+n x
[1,) 1 + n x


Z
n
d(x)
=
lim
2 2
[1,) n 1 + n x
Z
=
0d(x) = 0.
[1,)

1+n2 x2 dx = 2 .
x n
ex , and as cos( nx ) 1,
n)
n
Thus, 1 + nx
log(2+cos( nx ))

Therefore, we obtain limn

(b) Note that (1 +


we have log(2 +
x
cos( n )) log 3.
(log 3)ex , which
is clearly integrable. Therefore, by the dominated convergence theorem, we have

Z
Z 

 x 

 x 
x n
x n
lim
1+
log 2 + cos
dx =
lim
1+
log 2 + cos
dx
n 0
n
n
n
n
n
Z0
=
ex log(3)dx
0

=
4

log(3)

(c) Let us rewrite the integral as


Z n 
Z 
x
x
f 1 + 2 g(x)dx = lim
lim
f 1 + 2 [n,n] g(x) d(x).
n n
n R
n
n



Let M be such that |f (x)| < M for all x. Then f 1 + nx2 [n,n] g(x)
M |g(x)|, and since g L, we also have |g| L and thus M |g| L.
Therefore, by the dominated convergence theorem, we have
Z 
Z n 
x
x
f 1 + 2 g(x)dx = lim
f 1 + 2 [n,n] g(x) d(x)
lim
n R
n n
n
n
Z
 

x
=
lim f 1 + 2 [n,n] g(x) d(x)
n
n
ZR
=
f (1)R g(x)d(x)
R
Z
= f (1) g(x)d(x),
R

where the limit exists because f is continuous at 1.

Rt
2
2. Let f (t) = 0 ex dx. We will use DCT to evaluate limt f (t), even
without being able to evaluate the indefinite integral.
(a) Put h(t) = f (t)2 and g(t) =
g 0 (t).
(b) Show that h(t) + g(t) =

R1
0

et (1+x
1+x2

2)

dx. Show that h0 (t) =

for all t.
Z

(c) Use the previous parts to conclude that lim f (t) =


t

.
2

ex dx =

Solution. (a) Note that h0 (t) = 2f (t)f 0 (t) = 2et f (t), by the fundamental theorem of calculus.
h t2 (1+x2 ) i
t2 (1+x2 )

e
For g 0 (t), let f (x, t) = e 1+x2 , and note that t
=
1+x2
2

2tet (1+x ) = 2tet et x . Put (t) = 2tet . Note that 0 (t) =


2
(2 4t2 )et ,pand thus bypthe first derivative test, has a global
p
a global minimum of 2/e
maximum of 2/e at t = 1/2, and

p

2

at t = 12. Therefore, 2tet 2/e =: C for all t. Thus,

2 2
f
t Cet x C, which is clearly integrable on [0, 1]. Therefore,
by Theorem 5.9,
"Z
#
!
Z 1
2
2
1 t2 (1+x2 )
d
e
et (1+x )
dx
=
dx
dt 0
1 + x2
1 + x2
0 t
Z 1
2
2
=
2tet (1+x ) dx
0

=
=

2tet
2e

t2

0
t

et

x2

dx

eu du

2et f (t) = h0 (t),

as desired.
5

using the substitution u = tx

(b) As (h + g)0 = 0 for all t, we have that h + g is constant. Note that


R1 1

h(0) = 0. Moreover, g(0) = 0 1+x


2 dx = arctan(1) = 4 . Therefore,

(h + g)(0) = 4 , and as h + g is constant, the result follows.


1
(c) Note that |f (x, t)| 1+x
2 for all x, t, and thus by the dominated
convergence theorem, we have
Z 1
2
2
et (1+x )
dx
lim g(t) =
lim
t
1 + x2
0 t
Z 1
=
0 dx = 0.
0

But then limt h(t) =

4,

and thus limt f (t) =

/4 =

2 .

Extras:
5A. If f L(X, F, ) and a > 0, show that the set {x X | |f (x)| > a}
has finite measure. In addition, the set {x X | f (x) 6= 0} has -finite
measure.
R
Proof. As |f | L, we have that |f | d = A < . Let Ea = {x
X | |f (x)| > a}. Then we have |f | = |f |Ea + |f |Eac aEa + |f |Eac ,
and thus
Z
Z
A = |f | d a(Ea ) +
|f | d a(Ea ).
Eac

Therefore, (Ea ) A/a.


Moreover, {x X | f (x) 6= 0} = nZ+ En , and thus the set has -finite
measure.
5B. If f is a F-measurable real-valued function
and if f (x) = 0 for -almost
R
all x X, then f L(X, F, ) and f d = 0.
R
Proof. Note that |f | M + is -almost 0, and thus
|f | d
R
R = 0. This
f d |f | d = 0,
implies that
|f
|

L,
and
thus
f

L.
Moreover,
R
and thus f d = 0.
5D. If f L(X, F,
R ) and  > 0, then there exists a measurable simple function
such that |f | d < .
Proof. As f + and f are M + , we have measurable
simple functions
+
R
R
and such that + f + , f , and (f + + ) d < 2 , (f
) d < 2 . Let = + . Then we have is simple, and
Z
Z
Z

|f | d = |f + + (f )| d
|f + + | + |(f )| d < .

5F. If f belongs to L, it does not follow that f 2 belongs to L.


R
R1
Proof. Let f = 1x on [0, 1]. Then we have [0,1] f d = 0 1x dx =
1
2 x|0 = 2. However, f 2 = x1 does not have a finite integral on [0, 1], so
f2
/ L.
6

5G. Suppose
that f L(X, F, ), and that its indefinite integral is (E) =
R
f
d
for
E F. Show that (E) 0 for all E F if and only if
E
f (x) 0 for almost all x X. Moreover, (E) = 0 for all E if and only
if f (x) = 0 for almost all x X.
Proof. Write f = f + f . Suppose (E) > 0 for all E F. Let
 > 0, and Rput E = {x X | f (x) < }. As f is measurable, E F.
Moreover, E f d (E ) 0, and thus (E ) = 0. But then {x
X | f (x) < 0} = nZ+ E1/n has measure 0, and therefore f 0 -almost
everywhere.
On the other hand, if f 0 -almost everywhere,R then for all E F,
f E 0 -almost everywhere, and thus by 5B, f E d 0 for all
E F.
For the second piece, we repeat the above argument with E = {x
X | |f (x)| > }.
5H. Suppose that f1 , f2 L(X, F, ), and let 1 , 2 be their indefinite integrals. Show that 1 (E) = 2 (E) for all E F if and only if f1 (x) = f2 (x)
for almost all x X.
Proof. Note that 1 (E) = 2 (E) for all E if and only if (1 2 )(E) = 0
for all E, if and only if f1 f2 = 0 -almost everywhere (by problem
G).

You might also like